LSAT and Law School Admissions Forum

Get expert LSAT preparation and law school admissions advice from PowerScore Test Preparation.

User avatar
 Dave Killoran
PowerScore Staff
  • PowerScore Staff
  • Posts: 5853
  • Joined: Mar 25, 2011
|
#41287
Complete Question Explanation
(The complete setup for this game can be found here: lsat/viewtopic.php?t=15690)

The correct answer choice is (D)

The question stem creates a super-rule:
J95_Game_#1_#5_diagram 1.png
This powerful configuration naturally has a limited number of placement options, but those options are further limited by the fact that the two variables not involved in the super-rule—J and T—cannot be consecutive. Remember, look not just at the variables in a rule configuration, but also at the variables left out.

Consider how the interaction of J and T affects the super-rule:

  • If H, S, R, and K attempt to fill the first four slots, a violation occurs because J and T are consecutive in the last two slots (in whatever order).

    If H speaks first, and then S, R, and K attempt to fill the last three slots, a violation occurs because J and T are consecutive in the second and third slots (in whatever order).

    If H, S, R, and K attempt to fill the last four slots, a violation occurs because J and T are consecutive in the first two slots.

Consequently, only two placements of H, S, R, and K exist:

  • 1. H speaks first, and S, R, and K speak third, fourth, and fifth respectively:
J95_Game_#1_#5_diagram 2.png
  • 2. H speaks second, and S, R, and K speak fourth, fifth, and sixth respectively:
J95_Game_#1_#5_diagram 3.png
Accordingly, answer choice (D) could be true and is correct.
You do not have the required permissions to view the files attached to this post.
 medialaw111516
  • Posts: 80
  • Joined: Dec 11, 2018
|
#71080
Hi,

I always seem to have trouble with these questions when I'm trying to figure out where to place the block. Should I just try placing it in different positions and see? It always seems time consuming and I feel like there's a better way to do it and I must be missing something. I don't get how you decided where to place the SRK block.
 Jeremy Press
PowerScore Staff
  • PowerScore Staff
  • Posts: 1000
  • Joined: Jun 12, 2017
|
#71123
Hi medialaw,

This game on the whole is quite complex, and this question is a perfect illustration of the complexity. There's an interaction between two elements in the question that the original post is trying to describe for us. The first element is H :longline: SRK. That element is already awkward to fit. But it becomes even more awkward in conjunction with the second element: the fact that we have to keep J and T from being placed immediately next to one another.

Start by thinking about the most obvious placement of the H :longline: SRK element, with those variables simply filling the first four slots in that order. It won't work, because then J and T are smashed together onto 5 and 6. Thinking about that "most obvious" placement for H and SRK shows us the complexity of the question and what we have to keep an eye on: keeping J and T separate while still fitting the block plus H into the diagram.

Since SRK can't be placed at 2-3-4, shift it down and see if it will work at 3-4-5. Once you see you can make that full solution work (as in the original post), check the answers. If you see an answer that matches what you diagrammed (as you do here), you're done with the question. If you don't, you have to try the block out on 4-5-6 and see what that reveals.

Yes, it's trial and error, but it's the best you can do here without having a perfect vision of all the solutions to the game.

I hope this helps!

Jeremy
 hope
  • Posts: 84
  • Joined: Feb 13, 2018
|
#80210
To me, all answer choices are possible. Please show me how A B C and E violate anything.

A. HJSRKT
B. HTSRKJ
C. HTSRKJ
E. HTSRKJ

I see no violations in any of these answer choices, which means they can equally be true.

Help!
 Rachael Wilkenfeld
PowerScore Staff
  • PowerScore Staff
  • Posts: 1358
  • Joined: Dec 15, 2011
|
#80250
Hi Hope,

I think you may have overlooked the term "immediately" in some of these answer choices, or missed a rule..

For answer choice (A), if H is immediately before our SRK block, we have an issue with J and T. J cannot be first per the rules, but if T is first, J would either have to be next to T (a violation) or in the final spot (another violation.

Your solutions for answers B, C, and E all have J in the final slot, which is a violation of one of the stated rules.

For answer choice (B), if H is immediately before T, we end up with HT :longline: SRK. That would mean that J would have to be either before/after that long chain, which puts J first/last and violates our rules. Or, J would need to be between the HT and SRK portions of the sequence, but that would put J next to T, which is another no-no here.

For answer choice (C), if K is immediately before J, we have a sequence of H :longline: SRKJ. That only leaves us to place T. We can't place T after the sequence, because we'd have T and J next to each other. We can't put T anywhere else because it would push J to the sixth place.

For answer choice (E), we'd have the sequence H :longline: TSRK. We don't have to go any further with this one, because it has TSR consecutively, which is a violation of the rules.

Hope this helps, Hope.
Rachael
 hope
  • Posts: 84
  • Joined: Feb 13, 2018
|
#80266
Thank you Rachel. I get it now.

Get the most out of your LSAT Prep Plus subscription.

Analyze and track your performance with our Testing and Analytics Package.